Tricks to determine whether a given subset of R^n is a subspace or not.

แชร์
ฝัง
  • เผยแพร่เมื่อ 5 ก.ย. 2024
  • Join this channel to get access to perks:
    / @drmathaholic

ความคิดเห็น • 142

  • @aidanlee8567
    @aidanlee8567 11 หลายเดือนก่อน +15

    best video on subspaces, you deserve a shoutout

    • @DrMathaholic
      @DrMathaholic  11 หลายเดือนก่อน +2

      Thank you 😊

  • @musicattheendofmovies3702
    @musicattheendofmovies3702 ปีที่แล้ว +8

    you are the best person i have seen explain this concept so easy and precise i love it .

    • @DrMathaholic
      @DrMathaholic  ปีที่แล้ว +1

      Thank you.. glad to hear that🙏😊

  • @_flexion
    @_flexion ปีที่แล้ว +9

    Really well explained!! Thank you Sir

    • @DrMathaholic
      @DrMathaholic  ปีที่แล้ว +1

      Thank you and welcome 🙂

  • @deborah380
    @deborah380 3 หลายเดือนก่อน +1

    Thank you for doing examples with nonzero constants and powers other than 1!

  • @jimmyhunnicutt1923
    @jimmyhunnicutt1923 4 หลายเดือนก่อน +1

    you may have just saved my grade1 this is the first time I'm getting it, thank you!

    • @DrMathaholic
      @DrMathaholic  4 หลายเดือนก่อน +1

      Welcome.. all the best 👍

  • @BhagyashreeBhoomshetty
    @BhagyashreeBhoomshetty หลายเดือนก่อน +1

    Thank you sir❤
    Well explained 🙏

    • @DrMathaholic
      @DrMathaholic  หลายเดือนก่อน +2

      @@BhagyashreeBhoomshetty thank you ..welcome 😊

  • @LOLjerel
    @LOLjerel 4 หลายเดือนก่อน +1

    My professor is making us do both separately but this looks way easier lol Thank you!!

  • @georgehinneh6277
    @georgehinneh6277 7 หลายเดือนก่อน +1

    I love ❤ the hints you gave , it will be very helpful in mcq's

  • @lipansan
    @lipansan 7 หลายเดือนก่อน +1

    Thank you very much! Best explaning ever!

    • @DrMathaholic
      @DrMathaholic  7 หลายเดือนก่อน +1

      Thank you 😊

  • @whiteshadow5881
    @whiteshadow5881 ปีที่แล้ว +1

    W video, love it, saved it, i regret judging it because it used a chalkboard and came here last

    • @DrMathaholic
      @DrMathaholic  ปีที่แล้ว +1

      Glad to hear that..
      All the best😊

  • @gauravdateer9931
    @gauravdateer9931 ปีที่แล้ว +3

    Sir your explanation is tooo good 💯🔥

  • @REKHASHARMA14
    @REKHASHARMA14 หลายเดือนก่อน +1

    Thank you so much sir

    • @DrMathaholic
      @DrMathaholic  หลายเดือนก่อน +1

      @@REKHASHARMA14 welcome :)

  • @Arador1112
    @Arador1112 5 หลายเดือนก่อน +7

    tomorrow is my exam and I'm watching this now, nice explanation sir👍👍

    • @DrMathaholic
      @DrMathaholic  5 หลายเดือนก่อน +1

      All the best.. :)

  • @mrbleetoe
    @mrbleetoe ปีที่แล้ว +3

    You are the best, thank you!

    • @DrMathaholic
      @DrMathaholic  ปีที่แล้ว +1

      Thank you for your kind words 😊😊

  • @barakamtawa6571
    @barakamtawa6571 7 หลายเดือนก่อน +1

    thanks dr mathaholic am understand you very very good thanks🙏🙏

    • @DrMathaholic
      @DrMathaholic  7 หลายเดือนก่อน +2

      Very happy to hear that:)

  • @singh40506
    @singh40506 ปีที่แล้ว +2

    Thank you sir

  • @jayavaradhanp8405
    @jayavaradhanp8405 ปีที่แล้ว +6

    Sir make more videos on real analysis it will be helpful for net preparation.

    • @DrMathaholic
      @DrMathaholic  ปีที่แล้ว +1

      Yes sure..
      I will definitely try..

  • @saimamanzoor2781
    @saimamanzoor2781 ปีที่แล้ว +1

    Very very helpful vedio...thnkuu soo much

  • @mathematicsbya.g
    @mathematicsbya.g 7 หลายเดือนก่อน +1

    Thanks for the trick ❤

    • @DrMathaholic
      @DrMathaholic  7 หลายเดือนก่อน +1

      Welcome :)

  • @103anushkasingh3
    @103anushkasingh3 ปีที่แล้ว +1

    Very helpful video thanks sir!!😊

    • @DrMathaholic
      @DrMathaholic  ปีที่แล้ว +1

      Glad to hear that.. welcome 😊

  • @kukuyiehalem6391
    @kukuyiehalem6391 3 หลายเดือนก่อน +1

    "Show that W = {(x, y, z)|x = y and 2y = z} is a subspace of IR³." Sir is this a subspace?

    • @DrMathaholic
      @DrMathaholic  3 หลายเดือนก่อน +1

      Yes.. all terms of Linear and there is also no product of variables and there is no constant. So yes it's a subspace.
      Other way,
      W={ (x,x,2x) / x belongs to R}. Since y=x and z=2y=2x . So W is span of (1,1,2). So W is a subspace.

  • @mitra.1
    @mitra.1 ปีที่แล้ว +3

    Sir what about these 2. Will these form a subspace?
    1. {(x, y, z) : x + y = z}
    2. {(x, y, z) : x = y^2

    • @DrMathaholic
      @DrMathaholic  ปีที่แล้ว +5

      1 is a subspace.
      2 is not as it contains degree 2 term. For ex: (1,1) and (1,-1) satisfies the equation but their addition (2,0) does not satisfy the given equation. So addition fails..

  • @aishwariyajayan5100
    @aishwariyajayan5100 ปีที่แล้ว +1

    Very much thankful

  • @KWW9999
    @KWW9999 11 หลายเดือนก่อน +1

    You are a legend

  • @priyankas91
    @priyankas91 2 หลายเดือนก่อน +1

    Sir please explain this example:S={(x,y,z)/x≥y≥z},it is subspace or not

    • @DrMathaholic
      @DrMathaholic  2 หลายเดือนก่อน +1

      No,
      Take x= (3,2,1) and scalar c= -1.
      Then x vector is in the set S but c*x is not in S.
      So scalar multiplication fails.

  • @ecmrn
    @ecmrn 9 หลายเดือนก่อน +1

    so helpful!

    • @DrMathaholic
      @DrMathaholic  9 หลายเดือนก่อน +1

      Thank you 😊

  • @sturkyturky9792
    @sturkyturky9792 4 หลายเดือนก่อน +2

    Hello, was wondering if (x,y,z) in R^3, (4x - 9y)^2 = z^2 is a subspace, thank you

    • @DrMathaholic
      @DrMathaholic  4 หลายเดือนก่อน +2

      Nope. As degree is not 1 so not a subspace.
      Take (1,0,4) and (0,1,9), these 2 points satisfy given equation but their addition (1,1,13) does not satisfy the given equation.

    • @sturkyturky9792
      @sturkyturky9792 4 หลายเดือนก่อน +2

      @@DrMathaholic Shouldn't it be (0,-1,9) satisfies the equation? because if you use (0,1,9) you get -9^2 = 9^2, and then by using (0,-1,9) the addition is (1,-1,13) which satisfies: (4(1)-9(-1))^2 = 13^2 which becomes 4 + 9 = 13, 13 = 13?

    • @theresahfosuah3869
      @theresahfosuah3869 2 หลายเดือนก่อน +2

      @@sturkyturky9792since such an equation doesn’t satisfy all real numbers regarding closure under addition, it’s not a subspace

  • @craftmedia3448
    @craftmedia3448 9 หลายเดือนก่อน +1

    Sir
    I am confused on this question
    Show that the following subspace is corresponding to vector space R³
    The set of plane passing through the origin
    {(x,y,z) | ax+by+cz=0 where a,b,c is a scalar}

    • @craftmedia3448
      @craftmedia3448 9 หลายเดือนก่อน +1

      Wether it is subspace or not Sir

    • @DrMathaholic
      @DrMathaholic  9 หลายเดือนก่อน +1

      Try to show that 3 conditions are satisfied.
      1. This set contains zero vector (0,0,0) bcoz when u replace x,y,z by 0,0,0 it satisfies the equation ax+by+cz=0.
      2. Suppose (x1,y1,z1) and (x2,y2,z2) satisfies the equation ac+by+cz=0. Can you show that (x1+x2,y1+y2,z1+z2) also satisfies the equation?
      3. Suppose (x1,y1,z1) satisfies the equation. Can you show that alpha(x1,y1,z1) also satisfies the equation? Where alpha is any real number.

    • @DrMathaholic
      @DrMathaholic  9 หลายเดือนก่อน +1

      Let me know if u get stuck.

    • @DrMathaholic
      @DrMathaholic  9 หลายเดือนก่อน

      @craftmedia3448 it's a subspace..

    • @craftmedia3448
      @craftmedia3448 9 หลายเดือนก่อน +1

      @@DrMathaholic ok Sir
      I try

  • @lordbryann
    @lordbryann ปีที่แล้ว +1

    the vector satisfies the equation, ggggg thanks

  • @shwetashukla4402
    @shwetashukla4402 2 ปีที่แล้ว +1

    Thanku Sir🙏

  • @gameparty3704
    @gameparty3704 9 หลายเดือนก่อน +1

    a savior

  • @user-bt3me2vz1m
    @user-bt3me2vz1m 2 หลายเดือนก่อน +1

    how about this ,,,
    V =
    
    a 1
    b c
    
    : a, b ∈ R
    
    under the usual matrix operations.

    • @DrMathaholic
      @DrMathaholic  2 หลายเดือนก่อน +1

      No.
      Because this set does not contain the zero vector, which is zero matrix.

  • @fabnavc4691
    @fabnavc4691 2 ปีที่แล้ว +1

    Thank u sir

  • @isurudilshan7172
    @isurudilshan7172 ปีที่แล้ว +3

    x^2 + y^2 + z^2 = 0. Sir can this be a subspace ? Seems like the only vector in this set is (0,0,0). So isn't it enough to form a subspace. The problem is confusing me. Btw your video is help me a lot..

    • @DrMathaholic
      @DrMathaholic  ปีที่แล้ว +1

      This is a singleton set.. {0} is always a subspace. We call it a trivial subspace..

    • @isurudilshan7172
      @isurudilshan7172 ปีที่แล้ว +1

      @@DrMathaholic thank u sir..♥️

  • @rajilakshmi3566
    @rajilakshmi3566 ปีที่แล้ว +1

    {(a,b,c)}/a=b+c} please explain how it is subspace of R3

    • @DrMathaholic
      @DrMathaholic  ปีที่แล้ว +1

      Let (x,y,z) nd (a,b,c) belongs to given set. We have a=b+c and x=y+z. For addition, (x+a,y+b,z+c) we gave,
      x+a=y+z+b+c=y+b+z+c so (x+a,y+b,z+c) belongs to the set.
      Can you try for scalar multiplication?

  • @tusharpersai5790
    @tusharpersai5790 ปีที่แล้ว +1

    in that sin x example it is not a vector space since it does not satisfy the vector addition property, so there is no point of checking whether it is a subspace or not? am i right sir

  • @mustakeemsheikh1854
    @mustakeemsheikh1854 10 หลายเดือนก่อน +1

    W={(x,y,z)}€R3/(y2=0)} so sir wheather this is sub space or not please tell
    And you're explanation is awesome ❤

    • @DrMathaholic
      @DrMathaholic  10 หลายเดือนก่อน +1

      As y is a real number so y^2=0 implies that y=0. So ultimately the set is {(x,0,z) | x,z are real numbers}.
      This set is nothing but xz plane and hence its a subspace.

    • @DrMathaholic
      @DrMathaholic  10 หลายเดือนก่อน +2

      Thank you 😊

  • @jayavaradhanp8405
    @jayavaradhanp8405 ปีที่แล้ว +1

    How to check solution set of differential equation form a vector space or not

    • @DrMathaholic
      @DrMathaholic  ปีที่แล้ว +1

      Check this video..
      I have given the solution
      th-cam.com/video/M7eNGMD-YEc/w-d-xo.html

  • @ektakesharwani8115
    @ektakesharwani8115 2 ปีที่แล้ว +2

    Sir your explanation is very nice please provide some important questions of group theory related uppsc exam

    • @DrMathaholic
      @DrMathaholic  2 ปีที่แล้ว +1

      Thank you.. Well, I dont have any such collection but I am sure you will get it online... All the very best...

  • @mitra.1
    @mitra.1 ปีที่แล้ว +1

    Sir what about this one (x, y, z) : x = y = z . Will this form a subspace?

    • @DrMathaholic
      @DrMathaholic  ปีที่แล้ว +1

      Yes, it forms a subspace.

  • @gamerom9324
    @gamerom9324 2 ปีที่แล้ว +2

    Sir can you tell which playlist we have to follow for 1 st chapter and its basics for ode and MVC plz

    • @DrMathaholic
      @DrMathaholic  2 ปีที่แล้ว +1

      Hi
      Here is the Playlist for unit I.
      Enjoy:)
      th-cam.com/play/PLwaXU7G6UrbefJGT7WXm-ONSzcNoR4foC.html

    • @gamerom9324
      @gamerom9324 2 ปีที่แล้ว +1

      @@DrMathaholic tysm sir😁

    • @DrMathaholic
      @DrMathaholic  2 ปีที่แล้ว +1

      @@gamerom9324 :)

  • @khadkabaniya4545
    @khadkabaniya4545 ปีที่แล้ว +1

    Sir what about x^2+y^2-z^2=0. Will it form a subspace??

    • @DrMathaholic
      @DrMathaholic  ปีที่แล้ว +2

      No,
      (1,0,1) and (0,1,1) satisfy this equation but their addition is (1,1,2) which does not satisfy the given equation..
      So addition property fails..u, v satisfies but u+v do not

  • @ArvindKumar-cq8km
    @ArvindKumar-cq8km 18 วันที่ผ่านมา +1

    Noce

  • @tanvikadahiya3751
    @tanvikadahiya3751 2 ปีที่แล้ว

    Please explain sir....
    W = {( x, 2y, 3z) : x,y,z belongs to R}
    Will W be a subspace of R³

    • @DrMathaholic
      @DrMathaholic  2 ปีที่แล้ว +1

      Yes... because W is x(1,0,0)+y(0,2,0)+z(0,0,3)= span{(1,0,0), (0,2,0), (0,0,3) }...

    • @DrMathaholic
      @DrMathaholic  2 ปีที่แล้ว +1

      From above one can see that W=R^3.
      So yes, W is a space..

    • @tanvikadahiya3751
      @tanvikadahiya3751 2 ปีที่แล้ว

      Thank you sir

    • @DrMathaholic
      @DrMathaholic  2 ปีที่แล้ว

      @@tanvikadahiya3751 welcome..

  • @olivierdebruijn8704
    @olivierdebruijn8704 11 หลายเดือนก่อน +1

    hank you for the very nice explanation. Will this form a subspace: W1={(x,y)∈R^2 |xy≥0}⊂R^2?

    • @DrMathaholic
      @DrMathaholic  11 หลายเดือนก่อน +1

      Welcome..
      No, it won't.
      Take (0,1) and (-1,0) both are in W1 but when u add them, its (-1,1) which is not in W1 as product of 1 and -1 is not >=0

    • @olivierdebruijn8704
      @olivierdebruijn8704 11 หลายเดือนก่อน +1

      @@DrMathaholic thank you for your very fast respone! I have another question, is it correct that W3 ={(x,y,z)∈R3 | ax+by+cz=d} is a subspace in R3 for all values of a,b,c and d=0?

    • @DrMathaholic
      @DrMathaholic  11 หลายเดือนก่อน +1

      @olivierdebruijn8704 welcome
      .yes , when d is 0, its plane passing through origin. Hence it do forms a subspace..

  • @malaykhare1006
    @malaykhare1006 2 ปีที่แล้ว +1

    sir if there is a set C2(complex) over R2, then it will be a vector space right?

    • @DrMathaholic
      @DrMathaholic  2 ปีที่แล้ว +1

      R2 is not a field. So c2 over R2 do not have VS structure.
      C2 over R is VS

    • @malaykhare1006
      @malaykhare1006 2 ปีที่แล้ว +1

      @@DrMathaholic okay sir thank you

  • @ankitdadarwal192
    @ankitdadarwal192 ปีที่แล้ว +1

    If degree is same on both side then that is subspace

    • @DrMathaholic
      @DrMathaholic  ปีที่แล้ว +2

      I didn't get the question..
      Set S={ p(x) | deg(p(x)) = what?? }
      What is the set S?

  • @rishirajbehera7183
    @rishirajbehera7183 6 หลายเดือนก่อน +1

    x = 5y does it forms a subspace?

    • @DrMathaholic
      @DrMathaholic  6 หลายเดือนก่อน +1

      Yes...

    • @rishirajbehera7183
      @rishirajbehera7183 6 หลายเดือนก่อน +1

      Sir there's a question which I'm not able to understand. It says, "Explain why no list of six polynomials is linearly independent in P4?" @@DrMathaholic

    • @DrMathaholic
      @DrMathaholic  6 หลายเดือนก่อน +1

      @@rishirajbehera7183 see if this video helps::
      th-cam.com/video/qkRjSDJf-e4/w-d-xo.html

    • @DrMathaholic
      @DrMathaholic  6 หลายเดือนก่อน +1

      @rishirajbehera7183 If V is a space of dimension n then you take any subset with more than n elements, it will always be linearly dependent. Here P4 has dim 5, so any set with more than 5 elements will always be dependent.
      This is due to- A set B is a basis if and only if it is maximal linearly independent set.

    • @rishirajbehera7183
      @rishirajbehera7183 6 หลายเดือนก่อน +1

      Thank you so much sir :) @@DrMathaholic

  • @BirhanuDejen-qy3xd
    @BirhanuDejen-qy3xd ปีที่แล้ว +1

    You are uniqe go on and tank you

    • @DrMathaholic
      @DrMathaholic  ปีที่แล้ว +1

      Thank you and welcome 🙂

  • @ujjwalmahajan7581
    @ujjwalmahajan7581 2 ปีที่แล้ว +2

    Hello sir
    I am having my LA ESE on Monday, tell me some important concepts and questions so that I can pass the exam...please sir....I am kindaa scared :{
    I am from COEP 1st sem

    • @DrMathaholic
      @DrMathaholic  2 ปีที่แล้ว +2

      Be relax.. see the videos n also read the same topic from text book.
      After that see solved examples from text book and then try to solve problems from tutorial sheet.
      If you get stuck then ask me here.
      After this, if u get time then go to exercise problems from textbook

    • @ujjwalmahajan7581
      @ujjwalmahajan7581 2 ปีที่แล้ว +2

      @@DrMathaholic ok sir thank you for the guidance

    • @DrMathaholic
      @DrMathaholic  2 ปีที่แล้ว +2

      @@ujjwalmahajan7581 all the best..

    • @ujjwalmahajan7581
      @ujjwalmahajan7581 2 ปีที่แล้ว +1

      @@DrMathaholic :)

  • @user-zm7dr1be4b
    @user-zm7dr1be4b ปีที่แล้ว +1

    W
    ={ (x1,x2,x3)/x1=x2} is it a subspace of W?

  • @sruthisaravanan82sruthisar15
    @sruthisaravanan82sruthisar15 ปีที่แล้ว +1

    {(x,y,z) belongs to V: x+y+z=0} sir this is Subspace or Not

  • @srinathshrestha3899
    @srinathshrestha3899 ปีที่แล้ว +1

    s = {{x,y,z,} belongs to R^3 : x+y=0 or y-z=0}

    • @DrMathaholic
      @DrMathaholic  ปีที่แล้ว +1

      Yes, it forms a subspace.
      We have, x=-y and y=z.
      Thus, S={( x,-x,-x) } I.e. s is a 1 dimension subspace of R^3

    • @srinathshrestha3899
      @srinathshrestha3899 ปีที่แล้ว +1

      @@DrMathaholic sir ! This questions was in our mid sem exam , and the it's not a subspace because (1,-1,0) and (0,1,1) are counter example

    • @DrMathaholic
      @DrMathaholic  ปีที่แล้ว +1

      @Srinath Shrestha oh, it's "or" in between.. I thought its "and"

    • @DrMathaholic
      @DrMathaholic  ปีที่แล้ว +1

      @@srinathshrestha3899 Under the "or" condition, it's not a subspace..whereas under "and" it's a subspace..

    • @DrMathaholic
      @DrMathaholic  ปีที่แล้ว +1

      So whenever there is an or condition between more than 1 equation then it's never a subspace..

  • @top1malayalam145
    @top1malayalam145 2 ปีที่แล้ว +1

    Thank you so much sir

  • @Lastphoenix3
    @Lastphoenix3 ปีที่แล้ว +1

    Thank you sir

  • @fayadhpm
    @fayadhpm 2 ปีที่แล้ว +1

    Thank you sir